7 Plot (6,2) on the grid.



My teacher never thought us this and this was on my homework I give you 59 points if you give me a answer

7 Plot (6,2) On The Grid.My Teacher Never Thought Us This And This Was On My Homework I Give You 59 Points

Answers

Answer 1

Answer:Go to the 6 number on the bottom line.Then go up until you reach 2

Step-by-step explanation:
|      
|
|
|
|        .  
|______

Should look like that-ish

Answer 2

Answer:

Step-by-step explanation:

Put the point to the right 6 times and up two times


Related Questions

The following non-homogeneous Laplace equation (Poison equation) mod-
els the distribution of electrical potential when an outside charge is present:
122+2g=27一1.
Solve the equation subject to the following boundary conditions:
u(2,0)=u(2,2m)=0,
"(0,4) = u (27, y) = 0.

Answers

Now we need to apply the given boundary conditions to obtain the specific solution for u(x, y):

Boundary conditions in x-direction:
X(2) = X(27) = 0

Boundary conditions in y-direction:
Y(0) = Y(2m) = 0

How to solve the given Poisson equation with the provided boundary conditions.

1. Identify the Poisson equation and boundary conditions.
2. Use the method of separation of variables to solve the equation.
3. Apply the boundary conditions to obtain the specific solution.

Step 1: Identify the Poisson equation and boundary conditions
The given Poisson equation is:
Δu + 2g = 27 - 1,
where Δu is the Laplacian of the potential function u(x, y).

The provided boundary conditions are:
u(2, 0) = u(2, 2m) = 0,
u(0, y) = u(27, y) = 0.

Step 2: Use the method of separation of variables
We assume that the solution u(x, y) can be written as a product of two functions, one depending on x and the other depending on y, i.e., u(x, y) = X(x)Y(y).

Now, let's substitute this into the Poisson equation:
Δu + 2g = 27 - 1,
which becomes
(X''(x)/X(x) + Y''(y)/Y(y)) + 2g = 26.

Separate the variables:
X''(x)/X(x) = -Y''(y)/Y(y) - 2g = λ,
where λ is the separation constant.

This gives us two ordinary differential equations:
X''(x) = λX(x),
Y''(y) = -(λ + 2g)Y(y).

Step 3: Apply the boundary conditions
Now we need to apply the given boundary conditions to obtain the specific solution for u(x, y):

Boundary conditions in x-direction:
X(2) = X(27) = 0

Boundary conditions in y-direction:
Y(0) = Y(2m) = 0

Solving these equations with their respective boundary conditions will give us a specific solution for the potential function u(x, y). However, it is important to note that solving these equations involves solving eigenvalue problems and possibly infinite series expansions. The full solution process is quite involved and goes beyond the scope of this answer.

Nevertheless, I hope this outline of the solution method helps you understand the process of solving the Poisson equation with given boundary conditions.

Learn more about Poisson equation.

brainly.com/question/30410181

#SPJ11

Which of the following combinations of side lengths would NOT form a triangle with vertices X, Y, and Z?
A.
XY = 7 mm , YZ = 14 mm , XZ = 25 mm
B.
XY = 11 mm , YZ = 18 mm , XZ = 21 mm
C.
XY = 11 mm , YZ = 14 mm , XZ = 21 mm
D.
XY = 7 mm , YZ = 14 mm , XZ = 17 mm

Answers

The combinations of side lengths that would NOT form a triangle with vertices X, Y, and Z is 7 mm , YZ = 14 mm , XZ = 25 mm.

option A.

What are the possible lengths of triangle?

The lengths of triangle are determined base a given set of rules;

let a, b, and c be the side lengths of a triangle;

Based on the rules of side lengths of triangles, the sum of length a and b must be greater than c, or the sum of a and c must be greater than b or the sum of b and c must be greater than a.

For option A;

7 mm + 14 mm < 25 mm (this cannot be)

For option B;

11 mm + 18 mm > 21 mm (this will work)

For option C;

11 mm + 14 mm > 21 mm (this will work)

For option D;

7 mm + 14 mm > 17 mm (this will work)

Learn more about lengths of triangle here: https://brainly.com/question/24597672

#SPJ1

A telephone calling card company allows for $0.25 per minute plus a one-time service charge of $0.75. If the total cost of the card is $5.00, find the number of minutes you can use the card.

Answers

The number of minutes you can use the card is 9 minutes

Finding the number of minutes you can use the card.

From the question, we have the following parameters that can be used in our computation:

Allows for $0.25 per minute One-time service charge of $0.75.

Using the above as a guide, we have the following:

f(x) = 0.25x + 0.75

If the total cost of the card is $5.00, the number of minutes you is

0.25x + 0.75 = 5

So, we have

0.25x = 4.25

Divide by 0.25

x = 9

Hence, the number of minutes is 9

Read more about proportion at

https://brainly.com/question/1781657

#SPJ1

A triangle shaped table top with an area of 324 square inches has a base of 8x+4 inches and a height of 4x+2 inches. Given the area of a triangle is half of its base times height, what is a reasonable value of x in this situation?

Answers

If the area of a triangle is half of its base times height then the reasonable value of x is = 4.

A triangle-shaped table top with an area of 324 square inches has a base of 8x+4 inches and a height of 4x+2 inches. Given that the area of a triangle is half of its base times height, we can use the formula:

Area = (1/2) * base * height

Plug in the given values:

324 = (1/2) * (8x + 4) * (4x + 2)

Now, we need to solve for x. Follow these steps:

1. Multiply both sides of the equation by 2 to get rid of the fraction:

2 * 324 = (8x + 4) * (4x + 2)

2. Simplify the equation:

648 = (8x + 4) * (4x + 2)

3. Expand the equation by multiplying the terms in the parentheses:

648 = 32x^2 + 16x + 16x + 8

4. Combine like terms:

648 = 32x^2 + 32x + 8

5. Move all terms to one side of the equation to set it equal to zero:

32x^2 + 32x - 640 = 0

Now, we need to find a reasonable value of x. You can solve this quadratic equation using factoring, the quadratic formula, or a graphing calculator. Using a graphing calculator, we find that x is approximately 4.

Therefore, a reasonable value of x in this situation is 4.

Learn more about Triangle:

https://brainly.com/question/1058720

#SPJ11

Rachel and nicole are training to run a half marathon. rachel begins by running 30 minutes on the


tirst day of training. each day she increases the time she runs by 3 minutes. nicole's training follows the


function f(x) = 5x + 30, where x is the number of days since the training began, and f(x) is the time in


minutes she runs each day. what is the rate of change in minutes per day for the training program that


has the least rate of change?


rachel:


starting minutes:


increase in rate:


equation:


nicole:


starting minutes:


increase in rate:


equation:

Answers

The training program with the least rate of change is Rachel's, with an increase of 3 minutes per day.

Rachel:
Starting minutes: 30
Increase in rate: 3 minutes per day
Equation: f(x) = 3x + 30

Nicole:
Starting minutes: 30 (since f(0) = 5(0) + 30 = 30)
Increase in rate: 5 minutes per day
Equation: f(x) = 5x + 30

To find the training program with the least rate of change, we need to find the derivative of each equation and set it equal to zero:

f'(x) = 3 for Rachel's equation
f'(x) = 5 for Nicole's equation

Since 3 is less than 5, Rachel's training program has the least rate of change. Therefore, the rate of change in minutes per day for Rachel's training program that has the least rate of change is 3 minutes per day.

More on change rates: https://brainly.com/question/13740952

#SPJ11

Find the critical point and determine if the function is increasing or decreasing on the given intervals. y = x2 - 4x?, x>0 (Use decimal notation. Give your answer to three decimal places.) critical point c= _____

Answers

The critical point is c = 2, the function is decreasing on the interval 0 < x < 2, and increasing on the interval x > 2.

To find the critical point of the function y = x^2 - 4x, we first need to find its derivative, which represents the slope of the tangent line at any point on the curve.

The derivative of y with respect to x is:

y' = 2x - 4

Now, we need to find the critical points, which occur where the derivative is zero or undefined. In this case, the derivative is a polynomial, so it is never undefined. To find where it equals zero, we set y' equal to zero:

0 = 2x - 4

Solving for x, we get:

x = 4/2 = 2

So, the critical point is c = 2.

Now, we need to determine if the function is increasing or decreasing on the interval x > 0. To do this, we can analyze the sign of the derivative. If y' > 0, the function is increasing; if y' < 0, the function is decreasing.

For x > 2 (to the right of the critical point), the derivative y' = 2x - 4 is positive (since 2x > 4 when x > 2). Therefore, the function is increasing on the interval x > 2.

For x < 2 (to the left of the critical point), the derivative y' = 2x - 4 is negative (since 2x < 4 when x < 2). Therefore, the function is decreasing on the interval 0 < x < 2.

In summary, the critical point is c = 2, the function is decreasing on the interval 0 < x < 2, and increasing on the interval x > 2.

To learn more about critical point, refer below:

https://brainly.com/question/31017064

#SPJ11

For y=f(x) = x^4 - 7x + 5, find dy and Δy, given x = 5 and Δx=0.2.

Answers

The derivative of y=f(x) = x⁴ - 7x + 5 is dy/dx = 4x³ - 7. For x = 5 and Δx=0.2, dy = 1.986 and Δy = -54.5504.

Given the function y = f(x) = x⁴ - 7x + 5, we can find its derivative with respect to x using the power rule of differentiation:

dy/dx = d/dx(x⁴) - d/dx(7x) + d/dx(5) = 4x³ - 7

Now, we can use the given value of x = 5 and Δx = 0.2 to find the values of dy and Δy:

dy = (4x³ - 7) dx, evaluated at x = 5 and Δx = 0.2

dy = (4(5)³ - 7) (0.2) = 198.6 × 10^(-2)

This means that a small change of 0.2 in x results in a change of about 1.986 in y.

To find Δy, we use the formula:

Δy = f(x + Δx) - f(x)

Substituting x = 5 and Δx = 0.2, we get:

Δy = ((5 + 0.2)⁴ - 7(5 + 0.2) + 5) - (5⁴ - 7(5) + 5)

Simplifying this expression gives:

Δy = (122.4496 - 177) = -54.5504

This means that a small change of 0.2 in x results in a change of about -54.5504 in y.

To practice more questions about derivatives:

https://brainly.com/question/23819325

#SPJ11

Find the height of a cone with a diameter of 12m whose volume is 226m3. Use 3. 14, and round your answer to nearest meter

Answers

The height of a cone with a diameter of 12m whose volume is 226m³ is 6 meters.

The formula for the volume of a cone is

V = (1/3) * π * r^2 * h

where V is the volume, r is the radius, h is the height, and π is approximately equal to 3.14.

We know the diameter of the cone is 12m, which means the radius is 6m.

We also know that the volume of the cone is 226m^3.

Substituting these values into the formula, we get:

226 = (1/3) * π * 6^2 * h

Simplifying:

226 = (1/3) * 3.14 * 36 * h

226 = 37.68h

h = 226/37.68

h ≈ 6

Therefore, the height of the cone is approximately 6 meters.

Learn more about height of the cone at https://brainly.com/question/22246577

#SPJ11

X is a discrete random variable. The table below defines a probability distribution for X.


What is the expected value of X?

Answers

The expected value of x is given as follows:

E(X) = 1.6.

What is the mean of a discrete distribution?

The expected value of a discrete distribution is given by the sum of each outcome multiplied by it's respective probability.

The distribution for this problem is given as follows:

P(X = -7) = 0.2.P(X = -3) = 0.1.P(X = 3) = 0.4.P(X = 7) = 0.3.

Hence the expected value is given as follows:

E(X) = -7 x 0.2 - 3 x 0.1 + 3 x 0.4 + 7 x 0.3

E(X) = 1.6.

Missing Information

The table is given by the image presented at the end of the answer.

More can be learned about the expected value of a discrete distribution at https://brainly.com/question/27899440

#SPJ1

When dilating a figure, the scale factor determines whether or not the figure is reduced or enlarged. This number is a fraction or whole number. Can you tell me which one has which effect?



A. Fraction enlarges, whole number reduces



B. Whole number enlarges, fraction reduces



C. Both types of numbers enlarge



D. Both types of numbers reduce

Answers

B. Whole number enlarges, fraction reduces.

When a figure is dilated by a whole number, the image is enlarged by a factor of that whole number. For example, if a figure is dilated by a scale factor of 2, the image will be twice as large as the original.

On the other hand, when a figure is dilated by a fraction, the image is reduced. For example, if a figure is dilated by a scale factor of 1/2, the image will be half as large as the original.

To know more about fraction refer here:

https://brainly.com/question/10354322

#SPJ11

A garden bed is 4’ by 3’ and a 6’ layer of soil will be spread over the garden. A bag of soil contains 2ft3 of soil how many bags r needed

Answers

36 bags of soil are required to spread a 6 feet layer over a garden bed that is 4 feet by 3 feet.

How many bags of soil are required to spread a 4 feet layer over the garden bed?

Given information:

The garden bed has a length of 4 feet.

The garden bed has a width of 3 feet.

The layer of soil to be spread over the garden bed is 6 feet.

One bag of soil contains 2 cubic feet of soil.

To find the number of bags of soil required to spread a 6 feet layer over the garden bed, we need to calculate the volume of soil needed and then divide it by the volume of each bag of soil.

The volume of soil needed can be calculated by multiplying the length, width, and height (depth) of the soil:

Volume = length x width x depth

Volume = 4 feet x 3 feet x 6 feet

Volume = 72 cubic feet

This means we need a total of 72 cubic feet of soil to spread a 6 feet layer over the garden bed.

Next, we need to determine the number of bags of soil required. Since each bag contains 2 cubic feet of soil, we can divide the total volume of soil needed by the volume of each bag to get the number of bags required:

Number of bags = Volume of soil needed / Volume of each bag

Number of bags = 72 cubic feet / 2 cubic feet per bag

Number of bags = 36 bags

Therefore, 36 bags of soil are required to spread a 6 feet layer over a garden bed that is 4 feet by 3 feet.

Learn more about garden bed

brainly.com/question/26532420

#SPJ11

1. Sally wants to buy a pair of shoes for $12. 50 and a shirt for $23. 50. If 50 points the sales tax is 8. 25%, what will be the amount of the sales tax Sally's purchase?​

Answers

To calculate the amount of the sales tax on Sally's purchase, we first need to add the prices of the shoes and the shirt together. So, $12.50 + $23.50 = $36. Then, we need to calculate 8.25% of $36, which is done by multiplying 36 by 0.0825. That gives us a sales tax of $2.97. So, the amount of the sales tax on Sally's purchase is $2.97.

In summary, Sally wants to buy shoes for $12.50 and a shirt for $23.50, and the sales tax is 8.25% on a purchase of 50 points. The amount of the sales tax on Sally's purchase is $2.97. In order to calculate the sales tax, we added the prices of the items together and then calculated 8.25% of that total.

To know more about sales tax refer here

https://brainly.in/question/34781718#

#SPJ11

GEOMETRY HELP COSINE, SINE TANGENT

please help y’all i have no idea what i am doing

Answers

x = about 27.1 or 27.05

sinX / 13 = sin119/25

cross multiply

sinX(25) = sin(119)13

evaluate by using inverse sine to get an angle measure

sin^-1((sin119) x 13) / 25) = 27.05, 27.1*

Can someone PLEASE help me ASAP? It’s due today!! I will give brainliest if it’s done and correct.

Answers

The number of different sandwiches that can be created with two different meats is D. 6.

How to find the number of sandwiches ?

The number of different sandwiches that can be created with two different meats can be found by using the combination formula: nCr = n! / r!(n-r)!

In this case, we have 4 options for the first meat and 3 options for the second meat (since we cannot repeat the first meat). Therefore, the number of different sandwiches is:

4C2 = 4! / 2!(4-2)! = 6

So there are 6 different sandwiches that can be created with two different meats.

Find out more on sandwiches at https://brainly.com/question/23745118

#SPJ1

An octagonal pyramid has a height of 12 and a side length of 4.14. find the surface area of the pyramid.



please provide steps so i can understand how it works

Answers

The surface area of octagonal pyramid with height of 12 and a side length of 4.24 is 281.477 unit²

Height of octagonal pyramid = 12

Side length of octagonal pyramid = 4.14

The surface area of octagonal pyramid is  

SA = 2s²( 1 + √2) + 4sh

Here, s is side length of the octagonal pyramid = 4.14

h is height of the octagonal pyramid = 12

putting the values in the equation we get

SA = 2 × 4.14 ( 1 + √2 ) + 4 × 4.14 × 12

SA = 82.757 + 198.72

SA = 281.477

The surface area of octagonal pyramid is 281.477

To know more about surface area click here :

https://brainly.com/question/29298005

#SPJ4

Calculator Bookwork code: G24 EEPX not allowed The graph below shows line A and point P. Work out the equatic straight line that is parallel to line A and passes through poin ive your answer in the form y = mx + c, where m and care int fractions in their simplest forms. to task Y 6 S&MNE 5- 4 3- 2 1 -8 -7 -6 -5 -4 -3 -2 -1.0 PAN3456 -2- -3- -4- -5- -6+ 23 4 5 6 7 8 Watch video LG Line A x​

Answers

The equation of the line passing through point P and parallel to line A is

y = (5/7)x + 2

How to find the equation

Line A passed through points (0, -1) and (7, 4), hence equation of line passing through point (0, -1) and (7, 4).

find the slope of the line:

m = (y2 - y1) / (x2 - x1)

m = (4 - (-1)) / (7 - 0)

m = 5/7

use the point-slope form of the equation of a line with the point (0, -1):

y - (-1) = (5/7)(x - 0)

y + 1 = (5/7)x

y = (5/7)x - 1

Therefore, the equation of the line passing through the points (0, -1) and (7, 4) is y = (5/7)x - 1.

for a line parallel to line A passing through point P we change the intercept to give

y = (5/7)x + 2

Learn more about equation of a line at

https://brainly.com/question/18831322

#SPJ1

NEXT QUESTION >
Triangle HNR is shown where point K is the
centroid, KW = (2y — 8.9), KH
(2y 8.9), KH = (4.5w - 5.9),
KR = (0.5y + 3.2), KN = (5x – 5.2), KD = (9w
and KT = (7.1x – 11.8).
Z
W
H
K

Answers

Answer:

a b e

Step-by-step explanation:

Kelsey's favorite crackers are available in two different sizes. Which coupon should Kelsey use to pay the lower price per ounce for the crackers?

Answers

Using the coupon that offers a $0.50 discount on the larger package would yield the lowest price per ounce at $0.1875. Kelsey should use this coupon to get the best value for her favorite crackers.

Kelsey has two options when it comes to purchasing her favorite crackers, and she needs to determine which coupon will result in the lowest price per ounce. To make an informed decision, Kelsey should compare the price per ounce of both cracker sizes and apply the appropriate coupon accordingly.

First, Kelsey should find the price per ounce for each size by dividing the total price of the package by the total number of ounces in the package. For example, if the smaller package costs $2.00 and contains 8 ounces of crackers, the price per ounce would be $2.00 / 8 = $0.25 per ounce. Similarly, if the larger package costs $3.50 and contains 16 ounces, the price per ounce would be $3.50 / 16 = $0.21875 per ounce.

Next, Kelsey should determine the discount offered by each coupon and calculate the new price per ounce after applying the respective coupon. For instance, if one coupon provides a 10% discount on the smaller package, the new price per ounce would be $0.25 * (1 - 0.1) = $0.225 per ounce. If another coupon offers a $0.50 discount on the larger package, the new price per ounce would be ($3.50 - $0.50) / 16 = $0.1875 per ounce.

Finally, Kelsey should compare the adjusted price per ounce for both packages and select the coupon that results in the lowest price per ounce. In this example, using the coupon that offers a $0.50 discount on the larger package would yield the lowest price per ounce at $0.1875. Therefore, Kelsey should use this coupon to get the best value for her favorite crackers.

To know more about price, refer to the link below:

https://brainly.com/question/29099558#

#SPJ11

The equation m = 1. 4p represents the mass, m, in grams, of


p polished stones.

Answers

We know that the total mass of 5 polished stones would be 7 grams according to this equation

Sure! The equation m = 1.4p represents the mass, m, in grams, of p polished stones.

This means that if you know the number of polished stones, p, you can use this equation to calculate their total mass, m, in grams. For example, if you have 5 polished stones, you can plug in p = 5 and solve for m: m = 1.4 x 5 = 7 grams.

So the total mass of 5 polished stones would be 7 grams according to this equation.

To know more about mass refer here

https://brainly.com/question/19694949#

#SPJ11

Based on the equation, the number of grams which the mass increase for every 7 polished stones is: D. 9.8 g.

What is the slope-intercept form?

In Mathematics and Geometry, the slope-intercept form of the equation of a straight line is given by this mathematical equation;

y = mx + c

Where:

m represent the slope or rate of change.x and y are the points.c represent the y-intercept or initial value.

Based on the information provided above, a linear equation that models the mass in grams is given by;

y = mx + c

m = 1.4p

By substituting the given parameter (x = 7 polished stones), we have the following:

m = 1.4(7)

m = 9.8 g

Read more on slope-intercept here: brainly.com/question/7889446

#SPJ4

Missing information:

The question is incomplete and the complete question is shown in the attached picture.

An 8-inch-by-4-inch hole is cut from a
rectangular metal plate, leaving borders
of equal width x on all four sides. The
area of the metal that remains is 32 in².
The equation (8 + 2x)(4+2x) - 32 = 32
models the area of the plate. What is the
value of x, the frame width?

Answers

Answer:

2 inches

Step-by-step explanation:

The area of the metal plate can be calculated by subtracting the area of the hole from the area of the original plate. The area of the original plate is:

8 inches x 4 inches = 32 square inches

The area of the hole is:

8 inches x 4 inches = 32 square inches

So the area of the metal that remains is:

32 square inches - 32 square inches = 0 square inches

According to the equation given, we know that:

(8 + 2x)(4 + 2x) - 32 = 32

Expanding this equation we get:

32 + 16x + 8x + 4x^2 - 32 = 32

Simplifying and rearranging we get:

4x^2 + 24x - 32 = 0

Dividing both sides by 4 we get:

x^2 + 6x - 8 = 0

We can solve this quadratic equation by factoring:

(x + 4)(x - 2) = 0

So x = -4 or x = 2. Since the width of the frame cannot be negative, the only valid solution is x = 2.

Therefore, the frame width is 2 inches.

Find the equation of the parabola described. Find the two points that define the latus rectum, and graph the equation.


Vertex at (0,0); axis of symmetry the y-axis; containing the point (6,4).


What is the equation of the parabola? Find the two points that define the latus rectum.

Answers

The equation of the parabola is:

x = ay²

The two points that define the latus rectum are (±9/64, 4).

How to find the equation of the parabola?

The equation of the parabola with vertex at (0,0) and axis of symmetry the y-axis can be written in the form x = ay^2, where a is a constant. Since the parabola contains the point (6,4), we can substitute these values to solve for a:

6 = a(4²)

6 = 16a

a = 6/16 = 3/8

So the equation of the parabola is x = (3/8)y².

To find the two points that define the latus rectum, we need to determine the focal length, which is the distance from the vertex to the focus.

Since the axis of symmetry is the y-axis, the focus is located at (0, f), where f is the focal length. We can use the formula f = a/4 to find f:

f = a/4 = (3/8)/4 = 3/32

So the focus is located at (0, 3/32). The two points that define the latus rectum are the intersections of the directrix, which is a horizontal line located at a distance of f below the vertex, with the parabola. The directrix is located at y = -3/32.

To find the intersections, we can substitute y = ±(16/3)x^(1/2) into the equation of the directrix:

y = -3/32

±(16/3)[tex]x^(^1^/^2^)[/tex]= -3/32

x = 9/64

So the two points that define the latus rectum are (±9/64, 4).

Learn more about parabola

brainly.com/question/31142122

#SPJ11

In right triangle RST, ST = 5, RT = 12, and RS = 13. Find tan (s)

Answers

In right triangle RST, the value of tan (s) is 12/5.

To find tan(s), we first need to determine which side is opposite angle S and which side is adjacent to angle S.

In this case, RT is the side opposite angle S, and ST is the side adjacent to angle S. Since tangent (x) or tan(x) is defined as the ratio of the length of the opposite side to the length of the adjacent side, we can write the formula for tan(s) as follows:

tan(s) = (opposite side) / (adjacent side)

Now we can plug in the given side lengths to calculate the value of tan(s):

tan(s) = RT / ST
tan(s) = 12 / 5

Thus, tan(s) = 12 / 5.

Learn more about tangent here: https://brainly.com/question/13583559

#SPJ11

later, the teaching assistant in charless economics course gives him some advice. the teaching assistant says, based on past experience, working on 25 problems raises a students exam score by about the same amount as reading the textbook for 1 hour. for simplicity, assume students always cover the same number of pages during each hour they spend reading. given this information, in order to use his 4 hours of study time to get the best exam score possible, how many hours should he have spent working on problems and how many should he have spent reading? 1 hour working on problems, 3 hours reading 2 hours working on problems, 2 hours reading 3 hours working on problems, 1 hour reading 4 hours working on problems, 0 hours reading

Answers

From the margin gain formula, the number of hours he should have spent working on problems are equal to 2 hours and number of hours he should have spent reading are equal to other 2 hours. So, option (c) is correct one.

The meaning of ''margin'' is either the ''edge'' or the last unit and it is calculated by the incremental adjustment to the outcome, due to a unit change in the control variable. Marginal gain = Ratio of change in the outcome variable to the change in the control variable.

We have, total number of practice questions raises a students exam score

= 25

Along with practice questions the same amount as reading the textbook for 1 hour. In this problem, the outcome variable is the number of practice problems solved and the control variable is the number of hours Eric spent working on the practice problems. The above figure 1 table shows the total number of problems solved and time. First we determine the marginal gain of each hour Eric spent working on the practice questions. See the table present in above figures 2. Now, he has only 4 hours of study time for the best exam score as possible. We have to determine number of hours he should have spent reading and working on problems. It is assumed that students always cover the same number of pages during each hour they read the textbook so the advice provide by their teaching assistant that they can establish the relationship between time spent on reading the textbook and doing practice problems. The relation is below, 1 hour of reading the textbook = 15 practice problems solved. So, we can compare the the effectiveness of Eric's time spent on either working on practice problems or reading the textbook by using the table above figure 3. The decision rule for the optimal allocation of Eric's 4 hours of work is, If he can solve more than 15 practice problems in any of the 4 hours, then he should spend that particular hour working on the practice problems instead of reading the textbook. If not, then Eric should spend that hour to read the textbook instead.

For more information about margin gain , refer:

https://brainly.com/question/12231343#SPJ4

Complete question:

The table present in above figure 1 complete the question.

Eric is a hard-working college freshman. One Sunday, he decides to work nonstop until he has answered 100 practice problems for his math course. He starts work at 8:00 AM and uses a table to keep track of his progress throughout the day. He notices that as he gets tired, it takes him longer to solve each problem. later, the teaching assistant in charless economics course gives him some advice. the teaching assistant says, based on past experience, working on 25 problems raises a students exam score by about the same amount as reading the textbook for 1 hour. for simplicity, assume students always cover the same number of pages during each hour they spend reading. given this information, in order to use his 4 hours of study time to get the best exam score possible, how many hours should he have spent working on problems and how many should he have spent reading?

a) 1 hour working on problems, 3 hours reading

b) 2 hours working on problems, 2 hours reading

c) 3 hours working on problems, 1 hour reading

d) 4 hours working on problems, 0 hours reading

Integrate the function. 「 dx ,X> 6. Give your answer in exact form. x2x² - 36

Answers

To integrate the function ∫(x² - 36) dx, we first need to factor out the expression inside the parentheses:
∫(x² - 36) dx = ∫(x - 6)(x + 6) dx

We can then use the power rule of integration to find the antiderivative:
∫(x - 6)(x + 6) dx = (1/3)x³ - 6x + C, where C is the constant of integration.

Since the original problem states X > 6, we can evaluate the definite integral using these limits:
∫(x² - 36) dx from 6 to X = [(1/3)X³ - 6X] - [(1/3)(6)³ - 6(6)]
= (1/3)X³ - 6X - 68

Therefore, the answer in exact form is (1/3)X³ - 6X - 68.

To integrate the given function, first note the correct notation for the function: ∫(x^2)/(x^2 - 36) dx for x > 6.

To solve this, we can use partial fraction decomposition. The given function can be rewritten as:

∫(A(x - 6) + B(x + 6))/(x^2 - 36) dx

Solving for A and B, we find that A = 1/12 and B = -1/12. Now we rewrite the integral as:

∫[(1/12)(x - 6) - (1/12)(x + 6)]/(x^2 - 36) dx

Next, separate the two terms and integrate them individually:

(1/12)∫[(x - 6)]/(x^2 - 36) dx - (1/12)∫[(x + 6)]/(x^2 - 36) dx

Now, notice that the integrals are of the form ∫u'/u dx. The integral of this form is ln|u|. So we have:

(1/12)[ln|(x - 6)| - ln|(x + 6)|] + C

Using the logarithm property, we can rewrite the answer as:

(1/12)ln|((x - 6)/(x + 6))| + C

That is the exact form of the antiderivative for the given function.

Learn more about Integration here: brainly.com/question/18125359

#SPJ11

A 15 ft ladder leans against the side of a house. the bottom of the ladder is 7 ft away from the side of the house. find x

Answers

The distance between the tip of the ladder to the ground or the value of 'x' is 13.27 ft.

We know that the ladder is leaning on the wall and thus it makes a right-angle triangle, where:

the hypotenuse(h) is the length of the ladder,

the base(b) is the distance between the foot of the ladder and the bottom of the wall,

and the height(x) is the distance between the tip of the ladder to the bottom of the wall which we need to find.

As the question is on right angled triangle we can use the Pythagoras theorem to find the value of 'x':

[tex]Height^2 + Base^2 = Hypotenuse^2\\x^2 + b^2 = h^2[/tex]

Now we know that h= 15ft, and b=7ft.

Substituting the values in the above equation we get :

[tex]x^2 + 7^2 = 15^2\\x^2 = 225 - 49\\x = \sqrt{176}\\x= 13.27 ft.[/tex]

Therefore the distance between the tip of the ladder to the ground or the value of 'x' is 13.27 ft.

Learn more about the Pythagoras theorem at:

https://brainly.com/question/27997683

#SPJ4

A function f(x) = 3x^4 dominates g(x) = x^4. True False

Answers

The given statement "A function f(x) = 3x^4 dominates g(x) = x^4" is True, which means that as x gets larger, the value of f(x) will increase much more rapidly than the value of g(x).

The function f(x) = 3x^4 has a coefficient of 3 for the x^4 term.The function g(x) = x^4 has a coefficient of 1 for the x^4 term.

As x increases or decreases, the 3x^4 term in f(x) will grow faster or be larger in magnitude than the x^4 term in g(x). Since f(x) grows faster or has a larger magnitude than g(x), we can conclude that f(x) dominates g(x).

Therefore, the function f(x) = 3x^4 has a higher degree than g(x) = x^4

To know more about function click here:

https://brainly.com/question/28049372

#SPJ11

PLEASE HELP ( I CAN GIVE BRAINLIEST)

Answers

Answer:

x = √(6^2 + 18^2) = √(36 + 324) = √360

= 6√10

Answer:

[tex]x = 6\sqrt{10}[/tex]

Step-by-step explanation:

You can find the height using [tex]c^2 = a^2 + b^2[/tex] formula.

[tex](6\sqrt{2})^2 = 6^2 + b^2.[/tex]

[tex]b^2 = 72-36=36.[/tex]

[tex]b=6.[/tex]

You can find x using the same formula.

[tex]x^2 = 6^2 + 18^2 = 360.[/tex]

[tex]x = 6\sqrt{10}[/tex]

The volume of a cone is 2700π cm^3. The diameter of the circular base is 30. What is the height of the cone.

Answers

Answer: 36

Step-by-step explanation:

[tex]\frac{1}{3} \pi 15^{2} h=2700\pi \\75h=2700\\h=36[/tex]

5x−4<10give your answer as an improper fraction in its simplest form.

Answers

The value of x as an improper fraction in its simplest form is 14/5.

To solve the inequality 5x - 4 < 10, we need to isolate x on one side of the inequality. First, we add 4 to both sides:

5x - 4 + 4 < 10 + 4

5x < 14

Then, we divide both sides by 5:

5x/5 < 14/5

x < 2.8

Therefore, the solution to the inequality is x < 2.8. However, the question asks for the answer as an improper fraction in its simplest form. To convert 2.8 to an improper fraction, we multiply both the numerator and denominator by 10 to get rid of the decimal:

2.8 * 10 / 1 * 10 = 28 / 10

To simplify the fraction, we divide both the numerator and denominator by their greatest common factor, which is 2:

28 / 10 = 14 / 5

Therefore, the answer is 14/5.

Learn more about improper fraction at https://brainly.com/question/27847209

#SPJ11

The lengths of manufactured nails are distributed normally, with a mean length of 6cm, which has a standard deviation of 2mm. what is the length for which 98% of the nails will be longer?

Answers

Answer:

The length for which 98% of the nails will be longer is approximately 6.466 cm.

Step-by-step explanation:

First, we need to convert the units of the standard deviation to centimeters, since the mean is also given in centimeters. 2 mm is equal to 0.2 cm.

Next, we need to find the z-score that corresponds to the 98th percentile. We can use a standard normal distribution table or calculator to find this value. The z-score corresponding to the 98th percentile is approximately 2.33.

Finally, we can use the formula for a z-score to find the length of nail corresponding to this z-score:

z = (x - μ) / σ

where:

z = 2.33μ = 6 cmσ = 0.2 cm

Solving for x, we get:

2.33 = (x - 6) / 0.2x - 6 = 0.2 * 2.33x - 6 = 0.466x = 6.466

Therefore, the length for which 98% of the nails will be longer is approximately 6.466 cm.

Other Questions
6.7 times 10 to the power of 8 order of operations with scientific notation A magazine provided results from a poll of 15001500 adults who were asked to identify their favorite pie. Among the 15001500 respondents, 1414% chose chocolate pie and the margin of error was given as plus or minus 33 percentage points.a. What values do ^p^, ^q^, n, E, and p represent?b. If the confidence level is 9999%, what is the value of ? The function (job) of the carbohydrate chains attached to the cell membrane is:a:assist in diffusion.b: assist in cell to cell identification (name tag of the cell).c: maintain membrane fluidity. What is the magnitude of the force in coulomb's law when one of the charge is double How much will the monthly payment be for a new car priced at $29,950 if the current finance rate is 36 months at 3. 16%? Include financing the 8% TT&L and make a 25% down payment. I need the answer fast!! Give a Multi-Paragraph Explanation on what "Daylong Efforts to Repair Levee Fail" is about. Alice is going to her primary care physician to get vaccinated. Which is a reason that Alice should choose to get vaccinated?Vaccination reduces the chance of ever getting sick.Vaccination prevents pathogens from entering the body.Vaccination helps keep those who cannot be vaccinated safe.Vaccination increases the chance that pathogens will be effective. Are there any text features? Are they helpful? If so, how?Are there any text features? Are they helpful? If so, how? 5 unit 8 quiz SOTyrone is on the debate team and is preparing an argument against evolutionary theory. Which point should he include in his argument? A. There are clear stages of social development. B. Societies strive for and often achieve progress. C. Evolution is something that can only occur in nature. D. Industrialization brought unprecedented destruction in the 20th century. genes on a plasmid are transferred through a population of microbes, sometimes of different species, through horizontal gene transfer. if a scientist sequences a specific gene carried on the plasmid and compared the sequence of many different microbes, what pattern of relationships would be seen? A roller coaster has a cart with a mass of 150 kg and a track that spans 75 meters. what is the average velocity of the cart if it took 3 minutes to complete the track? Screenshot below i have no idea how to solve this T and I Construction is paving a rectangular-shaped parking lot with Hot Asphalt Mix. The parking lot is 100 yards by 44 yards and will have a 9-inch base. If the Hot Asphalt Mix has a density of 140 pounds per cubic foot, about how many tons of mix will be needed to pave the parking lot?A. ) 77B. ) 91C. ) 2079D. ) 2772 50 PONTS ASAP Triangle LMN has vertices at L(1, 4), M(1, 0), and N(3, 4) Determine the vertices of image LMN if the preimage is rotated 90 clockwise about the origin.L(4, 1), M(0, 1), N(4, 3)L(1, 4), M(1, 0), N(3, 4)L(4, 1), M(0, 1), N(4, 3)L(1, 4), M(1, 0), N(3, 4) MARKING BRAINLEIST IF CORRECT ASAP [tex] \underline{ \large{ \sf{ \purple{Question:-}}}}[/tex]Differentiate between executing the applet with appletviewer and HTML file.[tex] \\ \\ \\ [/tex]Thank You ! Helppppppp anyoneeee? The term group polarization refers to the tendency for two opposing factions to emerge within a group two opposing factions to emerge within a group A the prevailing opinion within a group to become more extreme after discussion the prevailing opinion within a group to become more extreme after discussion B a single leader to eventually dominate most groups a single leader to eventually dominate most groups C one or more group members to eventually have their opinions disregarded one or more group members to eventually have their opinions disregarded D democratic leadership to decrease as the length of meetings increases Find the component form of u + v given the lengths of u and v and the angles that u and v make with the positive x-axis. || 0 || = 3, = 5 || v || = 1, , u" In Scene 5, Marigold is upset when Midas turns her roses to gold. Midas says, "But they are worth a fortune. " Marigold replies, "They are ruined-as I fear are you. " What do these lines reveal about the two characters and how they differ?